2014 dxdy logo

Научный форум dxdy

Математика, Физика, Computer Science, Machine Learning, LaTeX, Механика и Техника, Химия,
Биология и Медицина, Экономика и Финансовая Математика, Гуманитарные науки




Начать новую тему Ответить на тему
 
 Теннисный турнир
Сообщение02.03.2012, 23:36 
Аватара пользователя


01/12/11

8634
N теннисистов сыграли турнир в один круг (каждый сыграл с каждым из остальных ровно один раз). Ни для кого не секрет, что в теннисе ничьих не бывает. Для каждого $1\le i\le N$ игрок под номером $i$ выиграл $x_i$ партий. Доказать, что необходимым и достаточным условием для того, чтобы нашлись три игрока А, Б и В такие, что А выиграл у Б, Б выиграл у В, а В выиграл у А, является

$\sum x_i^2<\frac{(N-1)N(2N-1)}{6}$
(Putnam Competition)


Упрощённый (сильно) вариант этой же задачи:
$N\ge 3$ теннисистов участвовали в турнире. Известно, что каждые два теннисиста сыграли между собой ровно один раз и не было ни одного теннисиста, проигравшего все встречи. Доказать, что найдутся теннисисты A, B, C такие, что A выиграл у B, B у C, C у A. (В теннисе ничьих не бывает.)
(Московская Математическая Олимпиада)

 Профиль  
                  
 
 Re: Теннисный турнир
Сообщение03.03.2012, 10:07 
Заслуженный участник
Аватара пользователя


03/12/11
640
Україна

(Оффтоп)

Я так подозреваю, что Putnam Competition - это некое соревнование, в котором участвует только один человек по фамилии Putnam :wink:
Перепишем неравенство так: $$\sum_{i=1}^N x_i^2 < \sum_{i=1}^N (i-1)^2. \eqno(1)$$ Необходимость следует из примера, когда любой игрок с большим номером выиграл у любого игрока с меньшим номером - тогда в $(1)$ имеется равенство и нетранзитивной тройки игроков нет.
Достаточность доказывается по индукции. При $N=3$ неравенство $(1)$ возможно только в случае $x_1=x_2=x_3=1$, т.е. вся тройка-требуемая. Допустим, что утверждение доказано для $N=n \geqslant 3$. Докажем его для $N=n+1$. Пусть А - игрок, выигравший наибольшее количество партий (или один из таковых). Не ограничивая общности, можно считать, что это игрок под номером $n+1$. Пусть $W$ - множество игроков, у которых А выиграл, а $L$ - которым проиграл. Очевидно, что $W$ непусто. Если множество $L$ пусто, то верно равенство $x_{n+1}=n$, вычитая квадрат которого из имеющегося неравенства $(1)$ для $N=n+1$ получим неравенство $(1)$ для $N=n$, и существование нужной тройки в оставшейся группе из $n$ игроков следует из уже доказанного утверждения для $N=n$. Если множество $L$ непусто, то взяв любого игрока В из $L$, получим, что он должен был проиграть какому-то игроку Б из $W$, иначе он выиграл бы больше партий, чем А (он выиграл по крайней мере у всех из $W$ и у А), что противоречит выбору А. Но это означает, что тройка А,Б,В - требуемая.

 Профиль  
                  
 
 Re: Теннисный турнир
Сообщение03.03.2012, 10:36 
Аватара пользователя


01/12/11

8634
Dave в сообщении #544760 писал(а):

(Оффтоп)

Я так подозреваю, что Putnam Competition - это некое соревнование, в котором участвует только один человек по фамилии Putnam :wink:

http://math.scu.edu/putnam/
http://amc.maa.org/a-activities/a7-prob ... ndex.shtml

 Профиль  
                  
 
 Re: Теннисный турнир
Сообщение03.03.2012, 10:55 
Заслуженный участник
Аватара пользователя


03/12/11
640
Україна

(Оффтоп)

Ktina в сообщении #544764 писал(а):
Dave в сообщении #544760 писал(а):
Я так подозреваю, что Putnam Competition - это некое соревнование, в котором участвует только один человек по фамилии Putnam :wink:

http://math.scu.edu/putnam/
http://amc.maa.org/a-activities/a7-prob ... ndex.shtml
Это была шутка, если кто не понял.

 Профиль  
                  
 
 Re: Теннисный турнир
Сообщение04.03.2012, 01:34 
Заслуженный участник
Аватара пользователя


03/12/11
640
Україна
А можно узнать, какого года эта задача? Я хочу посмотреть, как Putnam её решал. По вышеуказанным ссылкам такого условия не нашёл.

 Профиль  
                  
 
 Re: Теннисный турнир
Сообщение04.03.2012, 14:07 
Аватара пользователя


01/12/11

8634
Dave в сообщении #545048 писал(а):
А можно узнать, какого года эта задача? Я хочу посмотреть, как Putnam её решал. По вышеуказанным ссылкам такого условия не нашёл.

18-й Патнэм, 1958-ой год, задача B3.
http://150.19.10.56/www.kalva.demon.co.uk/putnam.html

 Профиль  
                  
Показать сообщения за:  Поле сортировки  
Начать новую тему Ответить на тему  [ Сообщений: 6 ] 

Модераторы: Модераторы Математики, Супермодераторы



Кто сейчас на конференции

Сейчас этот форум просматривают: нет зарегистрированных пользователей


Вы не можете начинать темы
Вы не можете отвечать на сообщения
Вы не можете редактировать свои сообщения
Вы не можете удалять свои сообщения
Вы не можете добавлять вложения

Найти:
Powered by phpBB © 2000, 2002, 2005, 2007 phpBB Group